LSAT and Law School Admissions Forum

Get expert LSAT preparation and law school admissions advice from PowerScore Test Preparation.

 Administrator
PowerScore Staff
  • PowerScore Staff
  • Posts: 8917
  • Joined: Feb 02, 2011
|
#71250
Complete Question Explanation

Cause and Effect, Principle, Justify. The correct answer choice is E.

Premise #1: Obesity will soon be the leading cause of preventable illness if current trends continue.

Premise #2: 45% of doctors don't feel qualified to advise their patients about weight.

Conclusion: Doctors need more training in the treatment of obesity.

The conclusion talks about training, but the premises don't address whether or not training would be helpful. Therefore, we want to look for the answer choice that justifies calling for more training.

The question stem asks which of the following principles most helps to justify the editorial's argument.

Answer Choice (A): The treatability of obesity is not mentioned in the premises as something that affects whether or not doctors need more training. This is putting unnecessary limitations on the conclusion.

Answer Choice (B): Again, we don't know how the treatment options for obesity changed, if they changed at all. This is not relevant to the argument.

Answer Choice (C): This answer choice does not mention training, and brings up a new idea about "doing good," which is not mentioned in the stimulus.

Answer Choice (D): Like answer choice (C), this answer doesn't mention training

Answer Choice (E): This is the correct answer choice. This answer choice is the only one which provides rationale for training doctors in the treatment of obesity. If doctors should be trained to treat the causes of preventable illnesses, and obesity is a cause of preventable illness (which we learn in the first sentence), then doctors should be trained to treat obesity.
 studyhelp20
  • Posts: 28
  • Joined: Dec 09, 2020
|
#82041
Can I please get some help with an explanation of the correct answer and how to diagram this question, if possible?
 Robert Carroll
PowerScore Staff
  • PowerScore Staff
  • Posts: 1787
  • Joined: Dec 06, 2013
|
#82155
s,

I don't see any point in diagramming this stimulus. The conditional in the first sentence doesn't really function in a way that makes diagramming useful - if that conditional were contraposed, or combined with other conditionals, or combined with other formal logic statements, or negated, I'd change my mind. It just seems to stand alone as speculation about what will happen if things continue without being changed.

The conclusion mentions training, but the premises don't talk about training at all. Nothing in the premises indicates that a lack of training is the cause of a problem - there is (arguably) a problem in the premises, but I don't know what its cause is. The author presents training as the solution to this problem. So I have a gap between the premises and conclusion, with "training" in the conclusion not being connected to the premises. I would look for an answer that supports the idea of training being a good idea, or imperative, or something similar.

With that in mind, answer choices (C) and (D) don't address training, so look quite unattractive. Answer choice (A) is a limitation on training - it says roughly "doctors should not be trained unless certain conditions are met." If we diagrammed that with the Unless Equation, we'd get something like the following:

doctors should be trained :arrow: there are effective treatments

This can't help. It says that training requires something else to be true. Well, either that thing is true, in which case...we get nothing (to think that supports training is to engage in a Mistaken Reversal). Or that thing is FALSE, in which case training is inappropriate - the opposite of what we want! This is a common property for a wrong answer in a Strengthen, Assumption, or Justify situation - the answer is a conditional that has the conclusion concept as a sufficient condition. That can never help the argument.

Answer choice (B) actually has the same problem. "Only if" is a necessary condition, so this answer choice is placing limitations on changing doctors' training. We want to support a change, not limit it.

Answer choice (E) tells us that doctors should be trained (what we're looking for in the conclusion!) to treat underlying causes of preventable illness (like obesity). So, for the kind of condition at issue, doctors should be trained! That definitely helps our conclusion.

Robert Carroll
User avatar
 landphil
  • Posts: 42
  • Joined: Jul 01, 2022
|
#102403
Robert Carroll wrote: Fri Dec 11, 2020 5:40 pm s,

I don't see any point in diagramming this stimulus. The conditional in the first sentence doesn't really function in a way that makes diagramming useful - if that conditional were contraposed, or combined with other conditionals, or combined with other formal logic statements, or negated, I'd change my mind. It just seems to stand alone as speculation about what will happen if things continue without being changed.

The conclusion mentions training, but the premises don't talk about training at all. Nothing in the premises indicates that a lack of training is the cause of a problem - there is (arguably) a problem in the premises, but I don't know what its cause is. The author presents training as the solution to this problem. So I have a gap between the premises and conclusion, with "training" in the conclusion not being connected to the premises. I would look for an answer that supports the idea of training being a good idea, or imperative, or something similar.

With that in mind, answer choices (C) and (D) don't address training, so look quite unattractive. Answer choice (A) is a limitation on training - it says roughly "doctors should not be trained unless certain conditions are met." If we diagrammed that with the Unless Equation, we'd get something like the following:

doctors should be trained :arrow: there are effective treatments

This can't help. It says that training requires something else to be true. Well, either that thing is true, in which case...we get nothing (to think that supports training is to engage in a Mistaken Reversal). Or that thing is FALSE, in which case training is inappropriate - the opposite of what we want! This is a common property for a wrong answer in a Strengthen, Assumption, or Justify situation - the answer is a conditional that has the conclusion concept as a sufficient condition. That can never help the argument.

Answer choice (B) actually has the same problem. "Only if" is a necessary condition, so this answer choice is placing limitations on changing doctors' training. We want to support a change, not limit it.

Answer choice (E) tells us that doctors should be trained (what we're looking for in the conclusion!) to treat underlying causes of preventable illness (like obesity). So, for the kind of condition at issue, doctors should be trained! That definitely helps our conclusion.

Robert Carroll
This is SOOO helpful Robert, thank you! This sums up a question I have had about a ton of strengthen questions that finally makes sense because of the way you put it.

Get the most out of your LSAT Prep Plus subscription.

Analyze and track your performance with our Testing and Analytics Package.